Smallest integer power for an inequality to hold












1












$begingroup$


so I have this inequality:
Given integers $m, kgeq1$.



$$2^{m/k} > frac{3}{2}$$



I'm interested in finding the smallest integer power $m$, as a function of $k$, that will make this inequality hold true.



I've made a few calculations and came up with $m$ as:
$$m(k) = [ 6k/10 + 0.5 ] - [k/70]$$



I'm almost certain that this $m$, for any given $k$, will satisfy the inequality. I believe it is the smallest integer (I don't know how to prove it, a proof will be appreciated!), but I wonder if there's a way to find another m that's a lot simpler... or maybe a way to simplify the m I found so I can use it in power additions and other things.



Or in general, is there a way/technique for such problems on how to find the smallest integer power?



Thank you for your time!










share|cite|improve this question











$endgroup$

















    1












    $begingroup$


    so I have this inequality:
    Given integers $m, kgeq1$.



    $$2^{m/k} > frac{3}{2}$$



    I'm interested in finding the smallest integer power $m$, as a function of $k$, that will make this inequality hold true.



    I've made a few calculations and came up with $m$ as:
    $$m(k) = [ 6k/10 + 0.5 ] - [k/70]$$



    I'm almost certain that this $m$, for any given $k$, will satisfy the inequality. I believe it is the smallest integer (I don't know how to prove it, a proof will be appreciated!), but I wonder if there's a way to find another m that's a lot simpler... or maybe a way to simplify the m I found so I can use it in power additions and other things.



    Or in general, is there a way/technique for such problems on how to find the smallest integer power?



    Thank you for your time!










    share|cite|improve this question











    $endgroup$















      1












      1








      1





      $begingroup$


      so I have this inequality:
      Given integers $m, kgeq1$.



      $$2^{m/k} > frac{3}{2}$$



      I'm interested in finding the smallest integer power $m$, as a function of $k$, that will make this inequality hold true.



      I've made a few calculations and came up with $m$ as:
      $$m(k) = [ 6k/10 + 0.5 ] - [k/70]$$



      I'm almost certain that this $m$, for any given $k$, will satisfy the inequality. I believe it is the smallest integer (I don't know how to prove it, a proof will be appreciated!), but I wonder if there's a way to find another m that's a lot simpler... or maybe a way to simplify the m I found so I can use it in power additions and other things.



      Or in general, is there a way/technique for such problems on how to find the smallest integer power?



      Thank you for your time!










      share|cite|improve this question











      $endgroup$




      so I have this inequality:
      Given integers $m, kgeq1$.



      $$2^{m/k} > frac{3}{2}$$



      I'm interested in finding the smallest integer power $m$, as a function of $k$, that will make this inequality hold true.



      I've made a few calculations and came up with $m$ as:
      $$m(k) = [ 6k/10 + 0.5 ] - [k/70]$$



      I'm almost certain that this $m$, for any given $k$, will satisfy the inequality. I believe it is the smallest integer (I don't know how to prove it, a proof will be appreciated!), but I wonder if there's a way to find another m that's a lot simpler... or maybe a way to simplify the m I found so I can use it in power additions and other things.



      Or in general, is there a way/technique for such problems on how to find the smallest integer power?



      Thank you for your time!







      algebra-precalculus inequality exponentiation integers






      share|cite|improve this question















      share|cite|improve this question













      share|cite|improve this question




      share|cite|improve this question








      edited Dec 2 '18 at 22:21









      Servaes

      23.3k33893




      23.3k33893










      asked Dec 2 '18 at 22:06









      Buk LauBuk Lau

      1878




      1878






















          2 Answers
          2






          active

          oldest

          votes


















          1












          $begingroup$

          Given a positive integer $k$ you want to find the least positive integer $m$ such that $2^{m/k}>frac{3}{2}$, or equivalently
          $$frac{m}{k}>frac{log(frac{3}{2})}{log2}=frac{log3-log2}{log2}=frac{log3}{log2}-1,$$
          or equivalently $m>kcdotleft(frac{log3}{log2}-1right)$. The latter is a constant; an online calculator tells me that
          $$frac{log3}{log2}-1approx0.5849625,$$
          so $m=lceil0.5849625ldotstimes krceil$.






          share|cite|improve this answer











          $endgroup$













          • $begingroup$
            Thank you for your quick answer! So m should be [k*0.5849625 + 0.5] correct?
            $endgroup$
            – Buk Lau
            Dec 2 '18 at 22:12





















          1












          $begingroup$

          HINT



          We have that since $log$ function is strictly increasing



          $$2^{m/k} > 3/2 iff log (2^{m/k}) > log (3/2)$$






          share|cite|improve this answer









          $endgroup$













            Your Answer





            StackExchange.ifUsing("editor", function () {
            return StackExchange.using("mathjaxEditing", function () {
            StackExchange.MarkdownEditor.creationCallbacks.add(function (editor, postfix) {
            StackExchange.mathjaxEditing.prepareWmdForMathJax(editor, postfix, [["$", "$"], ["\\(","\\)"]]);
            });
            });
            }, "mathjax-editing");

            StackExchange.ready(function() {
            var channelOptions = {
            tags: "".split(" "),
            id: "69"
            };
            initTagRenderer("".split(" "), "".split(" "), channelOptions);

            StackExchange.using("externalEditor", function() {
            // Have to fire editor after snippets, if snippets enabled
            if (StackExchange.settings.snippets.snippetsEnabled) {
            StackExchange.using("snippets", function() {
            createEditor();
            });
            }
            else {
            createEditor();
            }
            });

            function createEditor() {
            StackExchange.prepareEditor({
            heartbeatType: 'answer',
            autoActivateHeartbeat: false,
            convertImagesToLinks: true,
            noModals: true,
            showLowRepImageUploadWarning: true,
            reputationToPostImages: 10,
            bindNavPrevention: true,
            postfix: "",
            imageUploader: {
            brandingHtml: "Powered by u003ca class="icon-imgur-white" href="https://imgur.com/"u003eu003c/au003e",
            contentPolicyHtml: "User contributions licensed under u003ca href="https://creativecommons.org/licenses/by-sa/3.0/"u003ecc by-sa 3.0 with attribution requiredu003c/au003e u003ca href="https://stackoverflow.com/legal/content-policy"u003e(content policy)u003c/au003e",
            allowUrls: true
            },
            noCode: true, onDemand: true,
            discardSelector: ".discard-answer"
            ,immediatelyShowMarkdownHelp:true
            });


            }
            });














            draft saved

            draft discarded


















            StackExchange.ready(
            function () {
            StackExchange.openid.initPostLogin('.new-post-login', 'https%3a%2f%2fmath.stackexchange.com%2fquestions%2f3023280%2fsmallest-integer-power-for-an-inequality-to-hold%23new-answer', 'question_page');
            }
            );

            Post as a guest















            Required, but never shown

























            2 Answers
            2






            active

            oldest

            votes








            2 Answers
            2






            active

            oldest

            votes









            active

            oldest

            votes






            active

            oldest

            votes









            1












            $begingroup$

            Given a positive integer $k$ you want to find the least positive integer $m$ such that $2^{m/k}>frac{3}{2}$, or equivalently
            $$frac{m}{k}>frac{log(frac{3}{2})}{log2}=frac{log3-log2}{log2}=frac{log3}{log2}-1,$$
            or equivalently $m>kcdotleft(frac{log3}{log2}-1right)$. The latter is a constant; an online calculator tells me that
            $$frac{log3}{log2}-1approx0.5849625,$$
            so $m=lceil0.5849625ldotstimes krceil$.






            share|cite|improve this answer











            $endgroup$













            • $begingroup$
              Thank you for your quick answer! So m should be [k*0.5849625 + 0.5] correct?
              $endgroup$
              – Buk Lau
              Dec 2 '18 at 22:12


















            1












            $begingroup$

            Given a positive integer $k$ you want to find the least positive integer $m$ such that $2^{m/k}>frac{3}{2}$, or equivalently
            $$frac{m}{k}>frac{log(frac{3}{2})}{log2}=frac{log3-log2}{log2}=frac{log3}{log2}-1,$$
            or equivalently $m>kcdotleft(frac{log3}{log2}-1right)$. The latter is a constant; an online calculator tells me that
            $$frac{log3}{log2}-1approx0.5849625,$$
            so $m=lceil0.5849625ldotstimes krceil$.






            share|cite|improve this answer











            $endgroup$













            • $begingroup$
              Thank you for your quick answer! So m should be [k*0.5849625 + 0.5] correct?
              $endgroup$
              – Buk Lau
              Dec 2 '18 at 22:12
















            1












            1








            1





            $begingroup$

            Given a positive integer $k$ you want to find the least positive integer $m$ such that $2^{m/k}>frac{3}{2}$, or equivalently
            $$frac{m}{k}>frac{log(frac{3}{2})}{log2}=frac{log3-log2}{log2}=frac{log3}{log2}-1,$$
            or equivalently $m>kcdotleft(frac{log3}{log2}-1right)$. The latter is a constant; an online calculator tells me that
            $$frac{log3}{log2}-1approx0.5849625,$$
            so $m=lceil0.5849625ldotstimes krceil$.






            share|cite|improve this answer











            $endgroup$



            Given a positive integer $k$ you want to find the least positive integer $m$ such that $2^{m/k}>frac{3}{2}$, or equivalently
            $$frac{m}{k}>frac{log(frac{3}{2})}{log2}=frac{log3-log2}{log2}=frac{log3}{log2}-1,$$
            or equivalently $m>kcdotleft(frac{log3}{log2}-1right)$. The latter is a constant; an online calculator tells me that
            $$frac{log3}{log2}-1approx0.5849625,$$
            so $m=lceil0.5849625ldotstimes krceil$.







            share|cite|improve this answer














            share|cite|improve this answer



            share|cite|improve this answer








            edited Dec 2 '18 at 22:13

























            answered Dec 2 '18 at 22:09









            ServaesServaes

            23.3k33893




            23.3k33893












            • $begingroup$
              Thank you for your quick answer! So m should be [k*0.5849625 + 0.5] correct?
              $endgroup$
              – Buk Lau
              Dec 2 '18 at 22:12




















            • $begingroup$
              Thank you for your quick answer! So m should be [k*0.5849625 + 0.5] correct?
              $endgroup$
              – Buk Lau
              Dec 2 '18 at 22:12


















            $begingroup$
            Thank you for your quick answer! So m should be [k*0.5849625 + 0.5] correct?
            $endgroup$
            – Buk Lau
            Dec 2 '18 at 22:12






            $begingroup$
            Thank you for your quick answer! So m should be [k*0.5849625 + 0.5] correct?
            $endgroup$
            – Buk Lau
            Dec 2 '18 at 22:12













            1












            $begingroup$

            HINT



            We have that since $log$ function is strictly increasing



            $$2^{m/k} > 3/2 iff log (2^{m/k}) > log (3/2)$$






            share|cite|improve this answer









            $endgroup$


















              1












              $begingroup$

              HINT



              We have that since $log$ function is strictly increasing



              $$2^{m/k} > 3/2 iff log (2^{m/k}) > log (3/2)$$






              share|cite|improve this answer









              $endgroup$
















                1












                1








                1





                $begingroup$

                HINT



                We have that since $log$ function is strictly increasing



                $$2^{m/k} > 3/2 iff log (2^{m/k}) > log (3/2)$$






                share|cite|improve this answer









                $endgroup$



                HINT



                We have that since $log$ function is strictly increasing



                $$2^{m/k} > 3/2 iff log (2^{m/k}) > log (3/2)$$







                share|cite|improve this answer












                share|cite|improve this answer



                share|cite|improve this answer










                answered Dec 2 '18 at 22:08









                gimusigimusi

                92.8k84494




                92.8k84494






























                    draft saved

                    draft discarded




















































                    Thanks for contributing an answer to Mathematics Stack Exchange!


                    • Please be sure to answer the question. Provide details and share your research!

                    But avoid



                    • Asking for help, clarification, or responding to other answers.

                    • Making statements based on opinion; back them up with references or personal experience.


                    Use MathJax to format equations. MathJax reference.


                    To learn more, see our tips on writing great answers.




                    draft saved


                    draft discarded














                    StackExchange.ready(
                    function () {
                    StackExchange.openid.initPostLogin('.new-post-login', 'https%3a%2f%2fmath.stackexchange.com%2fquestions%2f3023280%2fsmallest-integer-power-for-an-inequality-to-hold%23new-answer', 'question_page');
                    }
                    );

                    Post as a guest















                    Required, but never shown





















































                    Required, but never shown














                    Required, but never shown












                    Required, but never shown







                    Required, but never shown

































                    Required, but never shown














                    Required, but never shown












                    Required, but never shown







                    Required, but never shown







                    Popular posts from this blog

                    Plaza Victoria

                    In PowerPoint, is there a keyboard shortcut for bulleted / numbered list?

                    How to put 3 figures in Latex with 2 figures side by side and 1 below these side by side images but in...